Vous êtes sur la page 1sur 7

U.A.E.

University, Department of Mathematical Sciences


Calculus for Business and Economics, Final Exam, Spring 2008
Name:

ID No:

Instructor's Name & Section:

Score:

/40

Part I. Di erentiation { Total 10 Points


1. (2 points) Find the derivative of the function. (Simpli cation is NOT required.)
(1) (1 point) f (x) = (8x 9) 3 .

Solution. By the Chain

Rule ,

we have

f 0 (x) = 3(8x


(2) (1 point) f (x) = ln 7x3

Solution. By the Chain


(7x3
0
f (x ) =

x2 + e

Rule ,

7x3

9)

3 1 (8x

9)0 = 3(8x 9) 4 (8) = 24(8x 9) 4 :

x2 .

we have

x2 )0
+e
x2

x2 (

x 2 )0

21x2 2x
= 3 2 +e
7x x

x2 (

2x) =

21x 2
x(7x 1)

2xe

x2 :

2. (2 points) Consider the equation 5y 2 8x4 + 3 = 0.


dy
(1) (1 point) Use the implicit differentiation to nd the derivative
from the given equation.
dx
Solution. We di erentiate the whole equation with respect to x:
d  2
5y
dx
dy
10y
dx

8 x4 + 3 =

d
(0) ;
dx

32x3 + 0 = 0;

d  2 d  4 d
d
5y
8x + (3) = (0)
dx
dx
dx
dx
3
dy
dy 32x
16x3
10y = 32x3 ;
=
=
:
dx
dx 10y
5y

dy
at the point (x; y ) = (1; 1).
dx
Solution. Putting (x; y) = (1; 1) into the result of (1) above, we get

(2) (1 point) Evaluate

dy
16x3
16
:

=

=
dx (x;y)=(1;1)
5y (x;y)=(1;1) 5

3. (3 points) Consider the function f (x) = 3x2 24x + 6.


(1) (2 points) Find all the critical values of f (x).

Solution. The critical value comes from the rst derivative of f (x):
f 0 (x) = (3x2

24x + 6)0 = 6x 24 = 6(x 4) = 0

at x = 4:

Hence, f (x) has only one critical value x = 4.

(2) (1 point) Determine the interval where f (x) is increasing and the interval where f (x) is decreasing.

Solution. We use the following sign chart:


x

Less Than 4

Sign of f 0 (x)
Graph of f (x)

Decreases

4
Bigger Than 4
f 0 (4) = 0
+
f (4) = 42
Increases

From this chart, we conclude that f (x) is increasing on (4; 1) and decreasing on (

1; 4).

4. (3 points) For the given functions, f (u) = u5 and g (x) = 2 3x2 , use the Chain Rule to compute the
derivative of (f  g ) (x), i.e., the derivative of f [g (x)].

Solution. The Chain

Rule

implies

(f [g (x)])0 = f 0 [g (x)]g 0 (x):

Since f 0 (u) = (u5 )0 = 5u4 and g 0 (x) = (2 3x2 )0 = 6x, so we have


(f [g (x)])0 = f 0 [g (x)]g 0 (x) = 5[g (x)]4 ( 6x) = 30x[g (x)]4 = 30x(2 3x2 )4 :

Part II. Applications { Total 12 Points


1. (3 points) A company estimates that it will sell N (t) hair dryers after spending t thousand dollars on
advertising. Explicitly N (t) is given as follows:
N (t) = 3t3 + 450t2

21600t + 1100;

40  t  60:

Find the interval where the rate of sales is increasing, i.e., N 0 (t) is increasing?

Solution. To determine the increment of a function f (x), we look for the sign of the derivative of f (x). In

this problem, we want to determine the increment of N 0 (t), so we use the sign of the derivative of N 0 (t), in
other words, the sign of N 00 (t).
N 0 (t) = 9t2 + 900t

21600;

N 00 (t) = 18t + 900 = 18(t

50) = 0

at t = 50:

We use the following sign chart (Be careful! The domain of N (t) is 40  t  60.):
Between 40 and 50
50
Between 50 and 60
Sign of N 00 (t)
+
N 00 (50) = 0
Graph of N 0 (t)
Increases
Decreases
t

From this chart, we conclude that N 0 (t) is increasing on (40; 50) and decreasing on (50; 60).

2. (2 points) How long will it take for 8400 Dhs to grow to 14600 Dhs at an interest rate of 9.4 percent if
the interest is compounded continuously ?

Solution. The ContinuouslyCompoundedInterest


14600 = 8400e0:094t ;

i:e:;

formula

e0:094t =

implies
14600 73
= :
8400 42

When we take the natural logarithmic function on both sides of the equation, we get
ln e0:094t

73 
= ln
;
42

i:e:;

73 
0:094t ln e = ln
;
42

where ln e = 1 is used.

i:e:;


1
73 
t=
ln
 5:88074 (years);
0:094
42

3. (7 points) The marketing research department for a company that manufactured and sells memory chips
for microcomputers established the following price{demand, revenue and const functions, respectively:
p(x) = 75

3x;

R(x) = x(75

3x);

C (x) = 125 + 16x;

where p(x) represents the whole sale price in Dirhams at which x millions chips can be sold. All functions
have domain 1  x  20.
(1) (1 point) Which one of (A) and (B ) below is a more plausible graph of the revenue function R(x)?

10

15

20

25

(B)
400

-100
300
-200
200
-300
100

(A)

-400

10

15

20

25

Solution. The revenue function R(x) = x(75 3x) = 3x2 + 75x is a quadratic function with the negative

constant 3 in front of x2 . Hence, the shape of the graph should be concave downward. One can use the
second derivative and the Second Derivative Test :
R0 (x) = 6x + 75;

R00 (x) = 6 < 0;

which implies that R(x) is concave downward by the Second Derivative Test . For these reasons, the
graph (B ) is better than (A).

(2) (3 points) Find the number of chips that will produce the maximum revenue. What is the maximum
revenue?

Solution. The revenue function is a quadratic function, which has the maximum/minimum value at the
vertex. So we nd the vertex either by completing the square or by using the derivative. Let us use the
latter method:
R(x) = x(75

3x) = 3x2 + 75x;

R0 (x) = 6x + 75 = 3(2x

25) = 0

at x =

25
= 12:5:
2

So R(x) has the critical value x = 25=2 at which R(x) has the value


25  1875
R
=
2
4

 468:75:

25 1875 
Thus, the quadratic function R(x) has the vertex
;
 (12:5; 468:75) and the vertex form
2 4
R (x ) =

3x2

+ 75x = 3 x

25 2 1875
:
+
2
4

Therefore, the number of chips producing the maximum revenue is x = 25=2 and the maximum revenue is
R (25=2) = 468:75.

(3) (1 point) What is the whole sale price per chip that produces the maximum revenue?

Solution. From the result in (2) above, the maximum revenue occurs at x = 25=2. The whole sale price
per chip is obtained by the pricedemand function p(x) = 75 3x and thus,

25  75
25 
= 75 3
= = 37:5
p
2
2
2


gives the whole sale price per chip producing the maximum revenue.
(4) (2 points) Find the break even points.

Solution. Break{even points occur when R(x) = C (x). So we solve the equation R(x) C (x) = 0 for x.
0 = R(x) C (x) = 3x2 + 75x (16x + 125) = 3x2 + 59x 125;

i:e:;

3x2

59x + 125 = 0:

The Quadratic Formula implies that the equation has the roots

59  1981
x=
6

 2:41526

or

17:2514:

Therefore, we have break{even points at x  2:41526 and x  17:2514.

Part III. Limits { Total 4 Points


x3 1
.
ln x
Solution. As x goes to 1, the denominator ln x goes to ln 1 = 0 and the numerator x3 1 goes to 13 1 = 0.
That is, we have the form 0=0. So by LHopitals Rule , we deduce

(4 points) Use LHopitals Rule to nd the limit: xlim


!1



x3 1
(x3 1)0
3x2
3 = 3:
lim
=
lim
=
lim
=
lim
3
x
x!1 ln x
x!1 (ln x)0
x!1 1=x
x!1

Part IV. Integration { Total 8 Points


1. (4 points) Compute
the integral.
Z


(1) (2 points) 5 t2 5t 2 dt.

Solution. A simple computation shows


Z

t2

5t 2 dt = 5

Z 
"

t2

t3
=5
3

5t 2 dt = 5
#

5t2
2

Z
"

t2 dt

t3
2t + C = 5
3

5 t dt
5t2
2

2 dt

2t + D;

where C and D are constants of integration.


(2) (2 points)

2
x

7ex

dx.

Solution. A simple computation shows


Z

2
x

7ex

dx =  2

Z
1
dx 7 ex dx =  2 ln jxj 7ex + C;
x

where C is the constant of integration.


2. (4 points) Use the IntegrationbySubstitution formula to nd the integral:

8x7 x8 + 4 dx.

Solution. Let u = x8 + 4. Then we get


du d  8 
=
x + 4 = 8 x7 ;
dx dx

i:e:;

du = 8x7 dx:

Thus, the given integral becomes


Z

8x7

x8

+ 4 dx =

u2
(x8 + 4)2
u du = + C =
+ C;
2
2

where C is the constant of integration.


6

Part V. Matrices and Linear Systems { Total 6 Points


1. (2 points) For matrices A and B given by
0

1 3
5 7

A=@

B=@

A;

2 4
6 8

1
A;

compute the product AB .

Solution. By the de nition of the matrix multiplication, we get


0

AB = @

1 3
5 7

10
A@

2 4
6 8

=@

2. (4 points) Consider the system:

1(2) + 3(6) 1(4) + 3(8)


5(2) + 7(6) 5(4) + 7(8)

8
<

=@

20 28
52 76

1
A:

2x + 3 y = 6
4x + 7 y = 8

X
(F)

When we introduce matrices A, X and C de ned by


0

A=@

2 3
4 7

X=@

A;

x
y

6
8

C=@

A;

1
A;

the given system (F) can be expressed as


0
@

2 3
4 7

10
A@

x
y

=@

6
8

1
A;

AX = C:

i:e:;

Multiplying by the inverse of A on both sides of the equation, we deduce


A 1 AX = A 1 C;

IX = A 1 C;

i:e:;

where I is the 2  2 identity matrix and A


(F), x and y , by computing A 1 C .

X = A 1 C;

i:e:;

is the inverse matrix of A. Find the solution of the system

Solution. By the de nition of the inverse matrix, we have


A

2(7) 3(4)

0
@

7
3
4 2

3
1 7
A= @
2
4 2

=@

7
2

3
2 A:

Putting this into X = A 1 C , we get


0
@

x
y

= X = A 1C = @

7
2

10

3
2 A@

6
8

1
A

7 (6)
@ 2

2(6) + 1(8)

Therefore, we deduce the solution of the system (F), x = 9 and y = 4.


7

3 (8)
2
A

=@

9
4

1
A:

Vous aimerez peut-être aussi